Đến nội dung

Hình ảnh

IMO 2013

imo

  • Please log in to reply
Chủ đề này có 27 trả lời

#1
Nesbit

Nesbit

    ...let it be...

  • Quản lý Toán Ứng dụng
  • 2412 Bài viết

*
Phổ biến

[Update] Vừa cập nhật đề thi của ngày thứ hai. Bên dưới là đề thi đầy đủ.

 

imo2013.gif

 

 

Ngày 1 (23/07/2013)

 

 

Bài 1. Chứng minh rằng với mọi số nguyên dương $k$ và $n$, tồn tại các số nguyên dương $m_1, m_2, \ldots, m_k$ sao cho \[ 1+\frac{2^k-1}{n}=\left(1+\frac{1}{m_1}\right)\left(1+\frac{1}{m_2}\right)\dots\left(1+\frac{1}{m_k}\right). \]


 

Bài 2. Trên mặt phẳng cho $2013$ điểm màu đỏ và $2014$ điểm màu xanh, trong đó không có ba điểm nào thẳng hàng. Ta chia mặt phẳng bởi các đường thẳng (không đi qua bất kì điểm nào trong các điểm đã cho) thành các vùng, sao cho không có bất kì vùng nào chứa các điểm có hai màu khác nhau. Hỏi cần ít nhất là bao nhiêu đường thẳng để luôn thực hiện được cách chia đó ?

 

 
Bài 3. Cho tam giác $ABC$ và $A_1$, $B_1$,  $C_1$ lần lượt là các điểm tiếp xúc của các đường tròn bàng tiếp với các cạnh $BC$, $AC$,  $AB$. Chứng minh rằng, nếu tâm đường tròn ngoại tiếp của tam giác $A_1B_1C_1$ nằm trên đường tròn ngoại tiếp của tam giác $ABC$, thì $ABC$ là tam giác vuông.
 
 
 
Ngày 2 (24/07/2013)
 
Bài 4. Cho tam giác nhọn $ABC$ có trực tâm $H$, và $W$ là một điểm trên cạnh $BC$, nằm giữa $B$ và $C$. Các điểm $M$ và $N$ theo thứ tự là chân các đường cao hạ từ các đỉnh $B$ và $C$. Gọi $\omega_1$ là đường tròn ngoại tiếp tam giác $BWN$, và $X$ là một điểm trên đường tròn sao cho $WX$ là đường kính của $\omega_1$. Tương tự, $\omega_2$ là đường tròn ngoại tiếp của tam giác $CWM$, và $Y$ là điểm sao cho $WY$ là đường kính của $\omega_2$. Chứng minh rằng ba điểm $X, Y$ và $H$ thẳng hàng. 

 
Bài 5. Cho $\mathbb Q_{>0}$ là tập hợp các số hữu tỉ dương, và $f: \mathbb Q_{>0} \to \mathbb R$ là hàm số thỏa mãn các điều kiện sau:

(i) $f(x)f(y) \geq f(xy)$ với mọi $x, y \in \mathbb Q_{>0}$,
(ii) $f(x+y) \geq f(x) + f(y)$ với mọi $x, y \in \mathbb Q_{>0}$,
(iii) Tồn tại số hữu tỉ $a> 1$ sao cho $f (a) = a$.

Chứng minh rằng $f(x) = x$ với mọi $x \in \mathbb Q_{>0}$.
 

Bài 6. Cho số nguyên $n\geq 3$ và xét $n+1$ điểm nằm cách đều nhau trên một đường tròn. Ta đánh số các điểm này bằng các giá trị $0,1,\dots, n$, không nhất thiết theo thứ tự, và hai điểm khác nhau thì được đánh hai số khác nhau. Hai cách đánh số được xem là như nhau nếu từ cách này có thể nhận được cách kia bằng cách xoay đường tròn. Một cách đánh số được gọi là đẹp nếu, với bất kì bốn số $a<b<c<d$ với $a+d=b+c$, dây cung nối các điểm được đánh số $a$ và $d$ không cắt dây cung nối các điểm được đánh số $b$ và $c$.

Gọi $M$ là số cách đánh số đẹp và $N$ là số các cặp số nguyên dương $(x,y)$ được sắp thứ tự (nghĩa là: $(x,y)$ và $(y,x)$ là khác nhau, trừ khi $x=y$) sao cho $x+y\leq n$ và $\gcd(x,y)=1$. Chứng minh rằng $M=N+1$.

 

 
 
Mời các bạn cùng thảo luận.

Bài viết đã được chỉnh sửa nội dung bởi Nesbit: 25-07-2013 - 14:43
dịch ẩu, đã sửa lại. Cảm ơn nguyenta

Không đọc tin nhắn nhờ giải toán.

 

Góp ý về cách điều hành của mod

 

 


#2
BlackSelena

BlackSelena

    $\mathbb{Sayonara}$

  • Hiệp sỹ
  • 1549 Bài viết

*
Phổ biến

Lời giải câu 3 ngày 1 của em và a Hân (perfectstrong)

PhotoShare(1).png

Gọi: $O$ là tâm của $(ABC)$, $O'$ là tâm của $(A_1B_1C_1)$ 

$A', B', C'$ lần lượt là trung điểm các cung $BAC, ABC, ACB$

$I_{B}$ là tâm đường tròn bàng tiếp $\angle B$, nó tiếp xúc $AB$ tại $L$

____________________

Dễ có $\triangle A'BC_1 = \triangle A'CB_1$ nên $A'C_1 = A'B_1$ tức $A'$ nằm trên trung trực của $B_1C_1$. Tương tự, ta cũng chỉ ra được $B'$ nằm trên trung trực $A_1C_1$ và $C'$ nằm trên trung trực của $A_1B_1 (*)$ 

Mặt khác, do $A_1,B_1,C_1$ lần lượt thuộc $BC,CA,AB$ nên $\triangle A_1B_1C_1$ nằm trong $(O)$

Nhưng theo giả thiết, $O'$ nằm trên $(O)$ nên $O'$ nằm ngoài $\triangle A_1B_1C_1$ tức $\triangle A_1B_1C_1$ tù.

Không giảm tính tổng quát, giả sử góc tù là góc $A_1$. Hiển nhiên $O'$ phải cùng phía với $A$ trên bờ $B_1C_1$. Từ $(*)$ ta có $O' \equiv A'$.

Lại có $\angle LAO' = \angle O'CB = \angle O'BC = \angle O'AC$

$\Rightarrow \triangle O'LA = \triangle O'B_1A \Rightarrow O'L = O'B_1$ hay $L \in (A_1B_1C_1)$

Sử dụng phương tích, ta có: $BC_1.BL = O'B^2 - O'A_1^2 \Leftrightarrow (p-a).p = O'B^2 - O'A_1^2$  $(1)$

Mặt khác, áp dụng định lý $\text{ Stewart}$ cho $\triangle O'BC$ với $A_1 \in \begin{bmatrix} BC \end{bmatrix}$:

$O'B^2.\overline{A_1C} + O'A_1^2.\overline{CB} + O'C^2.\overline{BA_1} + \overline{A_1C}.\overline{CB}.\overline{BA_1} = 0$

$\Leftrightarrow O'B^2.(p-b) - O'A_1^2.a + O'C^2.(p-c) - (p-b).(p-c).a = 0$

$\Leftrightarrow O'B^2.(p-b + p-c) - O'A_1^2.a = (p-b).(p-c).a$

$\Leftrightarrow O'B^2.a - O'A_1^2.a = (p-c).(p-b).a$

$\Leftrightarrow O'B^2 - O'A_1^2 = (p-b).(p-c)$  $(2)$

Từ $(1)$ và $(2)$ ta có $(p-a).p = (p-b).(p-c) \Leftrightarrow a^2 = b^2 + c^2 \Leftrightarrow Q.E.D$



#3
WhjteShadow

WhjteShadow

    Thượng úy

  • Phó Quản lý Toán Ứng dụ
  • 1323 Bài viết

*
Phổ biến

 

Hôm nay đã bước vào ngày thi đầu tiên.

 

 

 

 

Ngày 1 (23/07/2013)

 

 

Bài 1. Chứng minh rằng với mọi số nguyên dương $k$ và $n$, tồn tại các số nguyên dương $m_1, m_2, \ldots, m_k$ sao cho \[ 1+\frac{2^k-1}{n}=\left(1+\frac{1}{m_1}\right)\left(1+\frac{1}{m_2}\right)\dots\left(1+\frac{1}{m_k}\right). \]


 

 

 
 
Mời các bạn cùng thảo luận.

 

Sáng ngồi từ 6h30 đến 7h nghĩ ra bài 1 rồi mà phải đi học không kịp gõ :)) H mới được về trình bày.

Bài 1.

Có thể dễ dàng nhận thấy dạng bài này sử dụng quy nạp, và ta sẽ quy nạp the0 $k$

$\bullet$ Ta có khẳng định đề bài đúng với $k=1$ và mọi $n\in \mathbb{N}^{*}$. (Chọn $m_1=n$)

$\bullet$ Giả sử khẳng định đúng đến $k$ và đúng $\forall n\in \mathbb{N}^{*}$. Ta sẽ chứng minh khẳng định này đúng với $k+1$.

 Thật vậy để có điều này ta sẽ chứng minh 2 điều : Nếu đề bài đúng với bộ $(k;n)$ thì nó cũng đúng với $(k+1;2n)$ và đúng với $(k+1;2n-1)$.

+) Cm Nếu đề bài đúng với bộ $(k;n)$ thì cũng đúng với $(k+1;2n)$ :

\[ 1+\frac{2^k-1}{n}=\left(1+\frac{1}{m_1}\right)\left(1+\frac{1}{m_2}\right)\dots\left(1+\frac{1}{m_k}\right). \]

$$\Leftrightarrow 1+\frac{2^{k+1}-1}{2n}-\frac{1}{2n}=\left(1+\frac{1}{m_1}\right)\left(1+\frac{1}{m_2}\right)\dots\left(1+\frac{1}{m_k}\right)$$

$$\Leftrightarrow 1+\frac{2^{k+1}-1}{2n}=\left(1+\frac{1}{m_1}\right)\left(1+\frac{1}{m_2}\right)\dots\left(1+\frac{1}{m_k}\right)+\frac{1}{2n}$$

Mặt khác : $$\frac{2^{k}+n-1}{n}=\left(1+\frac{1}{m_1}\right)\left(1+\frac{1}{m_2}\right)\dots\left(1+\frac{1}{m_k}\right)$$

$$\Rightarrow \frac{1}{2n}=\frac{1}{2(2^{k}+n-1)}.\left(1+\frac{1}{m_1}\right)\left(1+\frac{1}{m_2}\right)\dots\left(1+\frac{1}{m_k}\right)$$

Vậy nên :

$$1+\frac{2^{k+1}-1}{2n}=\left(1+\frac{1}{m_1}\right)\left(1+\frac{1}{m_2}\right)\dots\left(1+\frac{1}{m_k}\right)+\frac{1}{2(2^{k}+n-1)}.\left(1+\frac{1}{m_1}\right)\left(1+\frac{1}{m_2}\right)\dots\left(1+\frac{1}{m_k}\right)$$

$$=\left(1+\frac{1}{m_1}\right)\left(1+\frac{1}{m_2}\right)\dots\left(1+\frac{1}{m_k}\right)\left(1+\frac{1}{2(2^{k}+n-1)}\right)$$

Chọn $m_{k+1}=2(2^{k}+n-1)$ ta có ngay đpcm.

+) Cm Nếu đề bài đúng với bộ $(k;n)$ thì cũng đúng với $(k+1;2n-1)$ :

\[ 1+\frac{2^k-1}{n}=\left(1+\frac{1}{m_1}\right)\left(1+\frac{1}{m_2}\right)\dots\left(1+\frac{1}{m_k}\right). \]

$$\Leftrightarrow 1+\frac{2^{k+1}-1}{2n-1}-\frac{n+2^{k}-1}{n(2n-1)}=\left(1+\frac{1}{m_1}\right)\left(1+\frac{1}{m_2}\right)\dots\left(1+\frac{1}{m_k}\right)$$

$$\Leftrightarrow 1+\frac{2^{k+1}-1}{2n-1}=\left(1+\frac{1}{m_1}\right)\left(1+\frac{1}{m_2}\right)\dots\left(1+\frac{1}{m_k}\right)+\frac{n+2^{k}-1}{n(2n-1)}$$

Mà :

$$\frac{n+2^{k}-1}{n(2n-1)}=\frac{1}{2n-1}.\left(1+\frac{1}{m_1}\right)\left(1+\frac{1}{m_2}\right)\dots\left(1+\frac{1}{m_k}\right)$$

Suy ra :

$$1+\frac{2^{k+1}-1}{2n-1}=\left(1+\frac{1}{m_1}\right)\left(1+\frac{1}{m_2}\right)\dots\left(1+\frac{1}{m_k}\right)+\frac{1}{2n-1}.\left(1+\frac{1}{m_1}\right)\left(1+\frac{1}{m_2}\right)\dots\left(1+\frac{1}{m_k}\right)$$

$$=\left(1+\frac{1}{m_1}\right)\left(1+\frac{1}{m_2}\right)\dots\left(1+\frac{1}{m_k}\right).\left(1+\frac{1}{2n-1}\right)$$

Chọn $m_{k+1}=2n-1$ ta cũng có ngay đpcm.

Vậy lời giải được hoàn tất (the0 nguyên lý quy nạp).

 

 

 

 

 

 


Bài viết đã được chỉnh sửa nội dung bởi WhjteShadow: 26-07-2013 - 08:37

“There is no way home, home is the way.” - Thich Nhat Hanh

#4
nguyenta98

nguyenta98

    Thượng úy

  • Hiệp sỹ
  • 1259 Bài viết

Bài 2. Trên mặt phẳng cho $2013$ điểm màu đỏ và $2014$ điểm màu xanh, trong đó không có ba điểm nào thẳng hàng. Ta chia mặt phẳng bởi các đường thẳng (không đi qua bất kì điểm nào trong các điểm đã cho) thành các vùng, sao cho không có bất kì vùng nào chứa các điểm có hai màu khác nhau. Hỏi cần ít nhất là bao nhiêu đường thẳng để luôn thực hiện được cách chia đó ?

 

Bài 1 lần này khá đơn giản, ý tưởng dùng quy nạp bổ dọc sau đó bổ ngang, (quy nạp mạnh dạng bảng)

Ý tưởng chung là quy nạp và rất giống như IMO 2012
Các bạn hãy cm hai nhận xét sau đây, khi đó bài toán sẽ dễ dàng được giải
NX1: $(i,k-1)$ đúng thì $(2i,k)$ đúng với $i\geq 1$
NX2: $(i+1,k-1)$ đúng thì $(2i+1,k)$ đúng với $i\geq 1$
Cm hai nhận xét này khá đơn giản, từ đó bài toán được giải xong bằng quy nạp mạnh

 

Bài 2: Dự đoán là $\left\lfloor\dfrac{2013+2014}{2}\right\rfloor$, giờ chỉ cần cm cái này thỏa mãn, chắc là quy nạp, và đã cm được $\left\lfloor\dfrac{2013+2014}{2}\right\rfloor$ là nhỏ nhất, thật vậy xét một cấu hình $2013+2014$ đỉnh như sau: Cho tất cả $2013+2013$ đỉnh đỏ và xanh này lên đường tròn , cách đều nhau, sao cho cứ một điểm xanh lại có một điểm đỏ đối xứng với nó qua tâm, xanh đỏ xen kẽ nhau, như vậy sẽ lấp đủ $2013+2013$ đỉnh, còn thừa một đỉnh xanh ta cho tùy ý vào vị trí giữa hai đỉnh kề nhau bất kì trong cấu hình trên, khi đó dễ thấy không có ba điểm thẳng hàng, số đường ít nhất là $\left\lfloor\dfrac{2013+2014}{2}\right\rfloor$, giờ cm nó là số thỏa mãn là xong


Bài viết đã được chỉnh sửa nội dung bởi nguyenta98: 24-07-2013 - 17:42


#5
Nesbit

Nesbit

    ...let it be...

  • Quản lý Toán Ứng dụng
  • 2412 Bài viết

Vừa cập nhật đề thi ngày thứ hai, mời các bạn cùng thảo luận. 

 

 

P/s


Không đọc tin nhắn nhờ giải toán.

 

Góp ý về cách điều hành của mod

 

 


#6
AM GM

AM GM

    Hạ sĩ

  • Thành viên
  • 54 Bài viết

 

[Update] Vừa cập nhật đề thi của ngày thứ hai. Bên dưới là đề thi đầy đủ.

 

imo2013.gif

 
 
Ngày 2 (24/07/2013)
 
Bài 4. Cho tam giác nhọn $ABC$ có trực tâm $H$, và $W$ là một điểm trên cạnh $BC$, nằm giữa $B$ và $C$. Các điểm $M$ và $N$ theo thứ tự là chân các đường cao hạ từ các đỉnh $B$ và $C$. Gọi $\omega_1$ là đường tròn ngoại tiếp tam giác $BWN$, và $X$ là một điểm trên đường tròn sao cho $WX$ là đường kính của $\omega_1$. Tương tự, $\omega_2$ là đường tròn ngoại tiếp của tam giác $CWM$, và $Y$ là điểm sao cho $WY$ là đường kính của $\omega_2$. Chứng minh rằng ba điểm $X, Y$ và $H$ thẳng hàng. 

 

 

 
 
Mời các bạn cùng thảo luận.

 

may em không có geo nen không vẽ đc hình mong cac anh thông cảm

đầu tiên gọi giao của 2 đtron la T ;CM đc AMTN;BNTW;TMCW nội tiếp

$\Rightarrow \widehat{NTA}= \widehat{NMA}=\widehat{ABC}= 180^{\circ}-\widehat{NTW}\Rightarrow A ;T;W$ thẳng hàng

lại có $\widehat{NXW}= \widehat{NBW}= \widehat{NHA}$ MÀ $\widehat{HNA}= \widehat{XNW}= 90^{\circ}\Rightarrow \Delta NHA$ ĐÒNG DẠNG $\Delta NXW\Rightarrow \Delta NXH$ đòng dạng $\Delta NWA$ SUY RA $\widehat{NXH}= \widehat{NWA}=\widehat{NWT}= \widehat{NXT}\rightarrow X;H;T$ THẢNG HÀNG

dễ dàng CM đc X ;T;Y thẳng hàng suy ra đpcm



#7
nguyenqn1998

nguyenqn1998

    Trung sĩ

  • Thành viên
  • 173 Bài viết

đáp án bài 5 http://www.artofprob...f=951&t=545437

đáp án bài 6 http://www.artofprob...f=951&t=545435



#8
nguyenta98

nguyenta98

    Thượng úy

  • Hiệp sỹ
  • 1259 Bài viết

 

Bài 2. Trên mặt phẳng cho $2013$ điểm màu đỏ và $2014$ điểm màu xanh, trong đó không có ba điểm nào thẳng hàng. Ta chia mặt phẳng bởi các đường thẳng (không đi qua bất kì điểm nào trong các điểm đã cho) thành các vùng, sao cho không có bất kì vùng nào chứa các điểm có hai màu khác nhau. Hỏi cần ít nhất là bao nhiêu đường thẳng để luôn thực hiện được cách chia đó ?

 

 

Mình thử giải, các bạn góp ý nhé

Giải như sau:

Trước tiên ta cm bổ đề 1: Cho $a$ điểm xanh và $a$ điểm đỏ cách đều nhau trên đường tròn và sắp xếp xen kẽ (tức một xanh lại một đỏ), thi số đường thẳng ít nhất để chia thành các vùng thỏa mãn điều kiện đề bài là $a$

Chứng minh:
Nếu ta coi một điểm xanh và một điểm đỏ kề nhau là một "cụm" thì tồn tại một đường thẳng đi qua giữa hai điểm xanh đỏ của cụm đó mà ta gọi tạm là đi qua cụm, khi đó do các cụm nằm trên đường tròn do đó một đường thẳng đi qua tối đa là hai cụm, mặt khác tổng số cụm là $a+a=2a$ do đó số đường thẳng ít nhất thỏa mãn là $2a/2=a$

Bổ đề 2: Hai điểm cùng màu bất kì trong các điểm đã cho có thể được cùng nằm trong một miền tạo bởi hai đường thẳng sao cho miền chứa hai điểm cùng màu đó ko chứa bất kì điểm khác màu nào còn lại

Chứng minh:
Xét hai điểm $A,B$ cùng màu giả sử là Xanh,  nối $A,B$ xét ở hai nửa mặt phẳng bờ $AB$ có hai điểm $X,Y$ màu đỏ sao cho $X$ gần $AB$ nhất và $Y$ gần $AB$ nhất ($X,Y$ ở hai nửa mặt phẳng khác nhau bờ $AB$) khi đó vẽ đường đường thẳng $//AB$ mà ta gọi là $s$ sao cho khoảng cách từ $s$ đến $AB$ nhỏ hơn khoảng cách từ $X$ đến $AB$ ($X$ không nằm trên $AB$ do ko có ba điểm thẳng hàng, lập tương tự về nửa mp bên kia $s'$ với $Y$

Từ đó $A,B$ nằm trong miền giữa hai đường thẳng $s,s'$ không chứa bất kì điểm đỏ nào khác, đpcm

Quay lại bài toán ở đây là $2013$ đỏ và $2014$ xanh, rõ ràng việc thêm một điểm xanh vào thì bổ đề trên kia vẫn đúng, như vậy số đường ít nhất bằng $\left\lfloor\dfrac{2013+2014}{2}\right\rfloor=2013$
Ta cm $2013$ đường thỏa mãn

Thật vậy xét bao lồi tất cả các điểm trên

TH1: Trong bao lồi (hay gọi tạm là đa giác lồi) tồn tại một đỉnh là đỉnh đỏ gọi là $C$

Khi đó xét hai đỉnh láng giềng với đỉnh đỏ đó (tức hai đỉnh nối với $C$ bằng cạnh của đa giác bao lồi) gọi chúng là $E,F$

Từ $C$ kẻ đường thẳng $l//EF$

Gọi $d_1$ là khoảng cách từ $EF$ đến $l$, $d_2$ là khoảng cách từ điểm xanh gần $C$ nhất

Khi đó vẽ $l'//l$ và $d(l'->l)<min\{d_1,d_2\}$ khi đó dễ thấy $l'$ chứa toàn điểm đỏ và nó chứa ít nhất một đỉnh đỏ

Do đó còn lại $2012$ đỉnh đỏ khác, áp dụng bổ đề 2 cho từng cặp đỉnh ta thu được $\dfrac{2012}{2}.2=2012$ đường

Cộng thêm $l'$ ta có tất cả $2012+1=2013$, chú ý do các đỉnh đỏ được cô lập và các miền chứa điểm đỏ ko chứa bất kì điểm xanh nào nên đó là một cách chia thỏa mãn, đpcm

TH2: Trong bao lồi, không có đỉnh đỏ tức là đỉnh của bao lồi toàn xanh

Khi đó gọi hai đỉnh xanh liên tiếp là $M,N$ và hai đỉnh láng giềng (với $M,N$ tương ứng) $U,V$
Gọi $d_1$ là khoảng cách từ $U$ đến $MN$, $d_2$ là từ $V$ đến $MN$ $d_3$ là khoảng cách của đỉnh đỏ gần nhất đến $MN$
Khi đó vẽ đường thẳng $k//MN$ và có khoảng cách đến $MN$ đủ nhỏ để nhỏ hơn $min\{d_1,d_2,d_3\}$ khi đó ở nửa mp bờ $k$ không chứa $U,V$ thì chứa toàn điểm xanh và chứa ít nhất 2 điểm xanh

Còn $2014-2=2012$ điểm xanh, thực hiện tương tự với bổ đề 2 ta có số đường là $\dfrac{2012}{2}.2+1=2013$ đây là đpcm

Vậy số đường ít nhất là $2013$

 

P/S bài 6 bộ sắp thứ tự $(x,y)$ mình tưởng nó có nghĩa là $(x,y)$ khác $(y,x)$ chứ nhỉ?


Bài viết đã được chỉnh sửa nội dung bởi nguyenta98: 25-07-2013 - 11:40


#9
Nesbit

Nesbit

    ...let it be...

  • Quản lý Toán Ứng dụng
  • 2412 Bài viết

P/S bài 6 bộ sắp thứ tự $(x,y)$ mình tưởng nó có nghĩa là $(x,y)$ khác $(y,x)$ chứ nhỉ?

Chết thật, dịch ẩu quá, đã sửa lại. Cảm ơn em nhiều !


Không đọc tin nhắn nhờ giải toán.

 

Góp ý về cách điều hành của mod

 

 


#10
gogo123

gogo123

    Trung sĩ

  • Thành viên
  • 102 Bài viết

Bài 4 có cách phương tích 

Gọi $L,L'$ là giao điểm của $AW$ với các đường trong (1) và (2) .

Phương tích suy ra $AL.AW=AN.AB$,$AL'.AW=AM.AC$ mà $AN.AB=AM.AC$ nên $L=L'$ hay A,L,W thẳng hàng .

Do $XY$ sông song với $O_1O_2$ (đường trung bình) nên $XY$ vuông góc với $AW$.

Gọi $H'$ là giao điểm cuả đường cao $AP$ với $XY$ suy ra 2 tam giác $AH'L$ và $AWP$ đông dạng.

Suy ra: $AH'.AP=AL.AW=AN.AB$ mà $AN.AB=AH.AP$ hay $H'=H$. Suy ra $X,Y,H$ thẳng hàng.

 

 

Hình gửi kèm

  • untitled.JPG

Bài viết đã được chỉnh sửa nội dung bởi gogo123: 25-07-2013 - 15:21

LKN-LLT


#11
ngoctruong236

ngoctruong236

    Trung sĩ

  • Thành viên
  • 146 Bài viết
Nguyênta98 oi phai la điểm Y ở xa AB nhất chứ,thế mới thỏa mãn


$\dpi{150}$ Giả sử ta có 2a +1 điểm màu đỏ và màu xanh điểm b, trong đó 2a+ 1 \leq b. Sau đó bởi trên SCP đối số, số lượng cần thiết của đường đến các điểm riêng biệt là 2a+ 1. Ngoài ra, rõ ràng là đủ số lượng dòng là 2a +2, bởi vì ta luôn luôn có thể sử dụng 1 cặp đường đến các điểm riêng biệt theo từng cặp. Từ vấn đề, chúng ta có thể kết luận rằng cho b = 2a +1 hoặc b = 2a+ 2, số lượng đầy đủ của dòng là 2a+ 1. Tuy nhiên, nó không phải là trường hợp đó cho mỗi b, số lượng đủ là 2a +1. Ví dụ, giả sử a = 1, b = 7. Ta có 2a +1 = 3 điểm màu đỏ và giả sử chúng ta đặt 7 điểm còn lại trong bảy khu vực khác nhau xác định bởi tam giác màu đỏ. Sau đó có ba dòng sẽ không đủ để điểm riêng biệt. Phần còn lại ai giúp mình với

Bài viết đã được chỉnh sửa nội dung bởi E. Galois: 26-07-2013 - 08:26


#12
nguyenta98

nguyenta98

    Thượng úy

  • Hiệp sỹ
  • 1259 Bài viết

Nguyênta98 oi phai la điểm Y ở xa AB nhất chứ,thế mới thỏa mãn

gần nhất bạn ạ, vì mình xét hai nửa mặt phẳng cơ mà :D



#13
ngoctruong236

ngoctruong236

    Trung sĩ

  • Thành viên
  • 146 Bài viết
12234.gif

gần nhất bạn ạ, vì mình xét hai nửa mặt phẳng cơ mà :D

minh nham thong cam nhe

#14
123123

123123

    Binh nhất

  • Thành viên
  • 21 Bài viết
Theo bạn WhjteShadow:

Sáng ngồi từ 6h30 đến 7h nghĩ ra bài 1 rồi mà phải đi học không kịp gõ :)) H mới được về trình bày.
Bài 1.

+) Cm Nếu đề bài đúng với bộ $(k;n)$ thì cũng đúng với $(k+1;2n+1)$ :


\[ 1+\frac{2^k-1}{n}=\left(1+\frac{1}{m_1}\right)\left(1+\frac{1}{m_2}\right)\dots\left(1+\frac{1}{m_k}\right). \]
$$\Leftrightarrow 1+\frac{2^{k+1}-1}{2n+1}-\frac{n+2^{k}-1}{n(2n+1)}=\left(1+\frac{1}{m_1}\right)\left(1+\frac{1}{m_2}\right)\dots\left(1+\frac{1}{m_k}\right)$$

$$\Leftrightarrow 1+\frac{2^{k+1}-1}{2n+1}=\left(1+\frac{1}{m_1}\right)\left(1+\frac{1}{m_2}\right)\dots\left(1+\frac{1}{m_k}\right)+\frac{n+2^{k}-1}{n(2n+1)}$$
Mà :
$$\frac{n+2^{k}-1}{n(2n+1)}=\frac{1}{2n+1}.\left(1+\frac{1}{m_1}\right)\left(1+\frac{1}{m_2}\right)\dots\left(1+\frac{1}{m_k}\right)$$
Suy ra :
$$1+\frac{2^{k+1}-1}{2n+1}=\left(1+\frac{1}{m_1}\right)\left(1+\frac{1}{m_2}\right)\dots\left(1+\frac{1}{m_k}\right)+\frac{1}{2n+1}.\left(1+\frac{1}{m_1}\right)\left(1+\frac{1}{m_2}\right)\dots\left(1+\frac{1}{m_k}\right)$$
$$=\left(1+\frac{1}{m_1}\right)\left(1+\frac{1}{m_2}\right)\dots\left(1+\frac{1}{m_k}\right).\left(1+\frac{1}{2n+1}\right)$$
Chọn $m_{k+1}=2n+1$ ta cũng có ngay đpcm.
Vậy lời giải được hoàn tất (the0 nguyên lý quy nạp).








Cho mình hỏi phần tô đỏ: Hình như phải là:$\Leftrightarrow 1+\frac{2^{k+1}-1}{2n+1}-\frac{2^{k}-1-n}{n\left ( 2n+1 \right )}=...$ mới đúng.
Nếu như vậy thì $m_{k+1}=2n+1$ không đúng.
Mình thấy:$(1+\frac{2^{k+1}-1}{2n+1}):(1+\frac{2^{k}-1}{n})= \frac{2^{k+1}n+2n^{2}}{2^{k+1}n+2n^{2}+2^{k}-n-1}$ chưa chắc lớn hơn 1 nên khó có thể viết được $1+\frac{2^{k+1}-1}{2n+1}=(1+\frac{2^{k}-1}{n})(1+\frac{1}{m_{k}})$ như thế.
Mong bạn xem lại hộ mình. Nếu mình sai sót, mong các bạn thông cảm.

Bài viết đã được chỉnh sửa nội dung bởi E. Galois: 26-07-2013 - 08:27


#15
ngoctruong236

ngoctruong236

    Trung sĩ

  • Thành viên
  • 146 Bài viết
$\dpi{150} \small Bài\, hinh \, cau 3\,minh \,giải \, thế này \, các \, bạn \,xem hộ \,mình \, \, \, \, \, \, \, \, \, \, \, \, \, \, \,Lấy \, G\,là \,tâm \,đường \,tròn \,ngoại \,tiêp \,\Delta A1B1C1 \,giả \,sử \,G \,nằm \,trên \,cung AB .\,Lấy \,M \,là \,điểm \,chính \,giữa \,cung AC \,\rightarrow MA=MC,AC1=CA1, \,lại \,có \,MACB là tứ \,giác \,nội \,tiếp\rightarrow \,\angle MAC1= \angle MCA1\rightarrow \Delta MC1A=\Delta MA1C\rightarrow MC1=MA1\rightarrow G\equiv M \, (\Delta A1B1C1)\cap BC\,tại X \,\Delta GXB va\Delta GC1B \,có \,GX=GC1 ,\,GB \,là \,cạnh \,chung \, Tứ\, giác\,GACB nt \rightarrow \,\angle GBX=\angle GAC=\angle GCA= \angle GBA \,\rightarrow \Delta GBA=\Delta GBX\rightarrow BX=BC1=s-a \,ta \, cũng\,có \,CA1=s-b \,va CX=CA=s \,.Tương\,tự \,(A1B1C1) \cap AC\,la \,Y \, ta \,cũng \,có \,AB1=CY=s-c \,và \,CB1=s-a. \,Theo \,tc \,phương \,tích \,ta \,có \,CA1. CX=CY.CB1\rightarrow s(s-b)=(s-a)(s-c) \rightarrow a^2=b^2+c^2\rightarrow dpcm\, \, \, \, \, \, \, \,$


IMO 3.JPG

#16
cleverboy

cleverboy

    Binh nhất

  • Thành viên
  • 25 Bài viết

Đây là một lời giải cho bài hình số 3 trên Mathlinks.ro

Hình gửi kèm

  • IMO 2013.png


#17
WhjteShadow

WhjteShadow

    Thượng úy

  • Phó Quản lý Toán Ứng dụ
  • 1323 Bài viết

Theo bạn WhjteShadow:


Cho mình hỏi phần tô đỏ: Hình như phải là:$\Leftrightarrow 1+\frac{2^{k+1}-1}{2n+1}-\frac{2^{k}-1-n}{n\left ( 2n+1 \right )}=...$ mới đúng.
Nếu như vậy thì $m_{k+1}=2n+1$ không đúng.
Mình thấy:$(1+\frac{2^{k+1}-1}{2n+1}):(1+\frac{2^{k}-1}{n})= \frac{2^{k+1}n+2n^{2}}{2^{k+1}n+2n^{2}+2^{k}-n-1}$ chưa chắc lớn hơn 1 nên khó có thể viết được $1+\frac{2^{k+1}-1}{2n+1}=(1+\frac{2^{k}-1}{n})(1+\frac{1}{m_{k}})$ như thế.
Mong bạn xem lại hộ mình. Nếu mình sai sót, mong các bạn thông cảm.

À cảm ơn bạn hôm đó mình nhầm :P Làm x0ng mình cứ trăn trở còn TH $n=1$ với mỗi $k$ thì sa0 :)) (Tại quy nạp như kia luôn bắt đầu từ $n=2$).

Có lẽ lời giải bây giờ chính xác rồi đó, bạn xem xem nhé !


“There is no way home, home is the way.” - Thich Nhat Hanh

#18
hungvu11

hungvu11

    Binh nhì

  • Thành viên
  • 13 Bài viết

Bài 5:

1. Ta sẽ chứng minh $f(x)$ tăng

Từ ii/ theo quy nạp ta có $f(na) \geq nf(a) = na$

Từ i/ ta có $f(n)f(a) \geq f(na) \geq na$ do đó $f(n) \geq n$ với mọi $n \in N^{*}$

Với $x \in Q^{+}$ tồn tại $n \in N^{*}$ sao cho $nx=m \in N^{*}$ 

Thay vào i/ ta có $f(n)f(x) \geq f(m)$ do đó $f(x) \geq 0$

Nên từ ii/ $f$ là hàm tăng

2. Chưng minh $f(x) = x$ với mọi $x \in N^{*}$

Từ i/ theo quy nạp ta có $ a^{n} \geq f(a^{n})$ với $n \in N^{*}$

Giả sử tồn tại $m \in N^{*}$ sao cho $f(m) = m +\varepsilon (\varepsilon >0)$

Khi đó tồn tại $N$ sao cho với $k>N$ thì $ k\varepsilon > m$

Do $ a>1$ nên $a^{n} \rightarrow \infty$ khi $n \rightarrow \infty$

Nên tồn tại $M$ sao cho  với $n>M$ thì $a^{n} \geq mN$

Khi đó cho $n>M$ tồn tại $p \in N^{*}$ sao cho $ 0 \leqslant a^{n} - pm < m$ với $p>N (1)$

Do f là hàm tăng nên $ a^{n} \geq f(a^{n}) \geq f(pm) \geq pm + p \varepsilon \geq pm + m $ do $p>N$ mâu thuẫn với $(1)$

Nên $f(m)=m$ với mọi $m \in N{*}$

Từ i/ và ii/ suy ra $mf(y) \geq f(my) \geq m.f(y)$

nên $mf(y)= f(my)$ với mọi $m \in N^{*}, y \in Q^{+}$

Từ đó suy ra $f(x)=x$ với mọi $x \in Q^{+}$.

 

 

 

 

 

 

 

 



#19
Nesbit

Nesbit

    ...let it be...

  • Quản lý Toán Ứng dụng
  • 2412 Bài viết

Thông tin cập nhật từ thầy Trần Nam Dũng:

 

Việc chấm thi IMO đã được bắt đầu. Do thứ tự chấm của các đoàn khác nhau nên hiện nay chưa thể bình luận gì nhiều. Việt Nam khá may mắn khi được bắt đầu từ bài toán 4, bài toán thuộc loại dễ của kỳ thi. Và chúng ta đã có 5 điểm 7 đầu tiên (bài của Trần Đăng Phúc chưa chấm xong). Như vậy là chí it ta đã có 5 cái Bằng khen (HR) rồi . Tất nhiên là chúng ta không chỉ chờ đợi như vậy.

 

 

BTC sẽ phô-tô bài làm của các thí sinh, gửi cho các trưởng đoàn. Trưởng pho đoàn tự chấm trước. Giám khảo chủ nhà cũng chấm (họ sẽ có phiên dịch của họ). Sau đó đến giờ đã định mình đến và thảo luân. Nếu điểm số 2 bên khớp thì chốt, nếu không thì sẽ tranh luận, họ sẽ hỏi chỗ này chỗ kia học sinh lý luận thế nào ... Cuối cùng đi đến thong nhất. Nói chung chấm thi quốc tế không quá khắt khe. Mỗi bài dù dễ hay khó nếu làm đúng đều được 7 điểm. Thường có 2 sơ đồ chấm là 0+ (làm chưa được, nhưng sẽ nhặt 1 số ý trung gian để cho điểm) và 7- (làm được nhưng sẽ trừ điểm ở những sai sót).

 

 

 

 

Thông tin cập nhật mới nhất, đoàn VN đã chấm xong bài 3: Công - 2, Đức - 7, Huy - 5, Kiên - * (chưa xong), Phúc - 0, Trung - 3. Như vậy Đức đang dẫn đầu với 14 điểm sau 2 bài.


Không đọc tin nhắn nhờ giải toán.

 

Góp ý về cách điều hành của mod

 

 


#20
25 minutes

25 minutes

    Thành viên nổi bật 2015

  • Hiệp sỹ
  • 2795 Bài viết

Thông tin bên Mathlinks thì China thứ $1$, Korea thứ $2$, USA thứ $3$ và Việt Nam thứ $7$

Top b1d5781111d84f7b3fe45a0852e59758cd7a87e5 countries are
1)China
2)Korea
3)USA
4)Russia
5)Korea(North)
6)Singapore
7)Vietnam
8)Taiwan
9)England
10)Iran


Hãy theo đuổi đam mê, thành công sẽ theo đuổi bạn.



Thảo luận BĐT ôn thi Đại học tại đây






Được gắn nhãn với một hoặc nhiều trong số những từ khóa sau: imo

4 người đang xem chủ đề

0 thành viên, 4 khách, 0 thành viên ẩn danh